3
$\begingroup$

Let us suppose that $X_1,\ldots,X_n$ with $n\ge1$ are iid random variables such that $\operatorname EX_1=0$ and $\operatorname E|X_1|^s<\infty$ with some $s>2$ and define the DFT of $X_1,\ldots,X_n$ by setting $$ D_n(\omega)=n^{-1/2}\sum_{t=1}^nX_te^{-it\omega} $$ for $n\ge1$ and $\omega\in[-\pi,\pi]$, where $i=\sqrt{-1}$. I am interested in the asymptotic behaviour of the expected value of the maximum of the periodogram given by $$ \operatorname E\max_{1\le j\le q}|D_n(\omega_j)|^2, $$ where $q=\lfloor(n-1)/2\rfloor$ and $\omega_j=2\pi j/n$ for $1\le j\le q$. If we also assume that $X_1,\ldots,X_n$ are Gaussian, then $\operatorname E\max_{1\le j\le q}|D_n(\omega_j)|^2=O(\log n)$ as $n\to\infty$ since $|D_n(\omega_1)|^2,\ldots,|D_n(\omega_q)|^2$ are iid standard exponential random variables (we can use the idea from this answer to establish the growth rate). I suspect that this might be true even if we do not assume Gaussianity. Intuitively, for large values of $n$, the distribution of $D_n(\omega)$ should be close to the Gaussian distribution.

Is it possible to establish that $\operatorname E\max_{1\le j\le q}|D_n(\omega_j)|^2=O(\log n)$ as $n \to\infty$ if $X_1,\ldots,X_n$ are iid random variables with zero means and finite moments of order $s>2$?

Any help is much appreciated!

$\endgroup$
5
  • $\begingroup$ Looks like it is, indeed, true though the computation is somewhat cumbersome. Let me know if you are still interested and I'll try to post it (if it survives the morning scrutiny) $\endgroup$
    – fedja
    Sep 9, 2018 at 0:28
  • $\begingroup$ @fedja I'm very interested! I'd greatly appreciate if you could post it. $\endgroup$
    – Cm7F7Bb
    Sep 9, 2018 at 8:30
  • $\begingroup$ I posted a sketch. Feel free to ask questions if something is unclear. $\endgroup$
    – fedja
    Sep 10, 2018 at 1:20
  • $\begingroup$ I noticed that you posted a few questions in a comment to my post but before I could get to answering them, they disappeared. Does it mean that you figured them out yourself? $\endgroup$
    – fedja
    Nov 21, 2018 at 14:20
  • $\begingroup$ @fedja Thank you very much for your response! Yes, I’ve made some progress and my questions were not relevant anymore. I’m reading the proof carefully and trying to understand every single bit of it. $\endgroup$
    – Cm7F7Bb
    Nov 21, 2018 at 14:25

1 Answer 1

3
$\begingroup$

Here is a sketch. Feel free to ask for clarifications if my writing gets too terse or confusing in places :-).

First recall the Bernstein (a.k.a. Hoeffding, Chernov, etc.) bound. If $Y_m$ are mean $0$ independent random variables bounded by $s$, then for $Y=\sum_{m=1}^n Y_m$, we have for every positive $t$, $$ P(|Y|\ge t)\le Ce^{-c\frac{t^2}{ns^2}}\,. $$ with some $C,c>0$. The proof goes via the consideration of $Ee^{\beta Y}$ with appropriately chosen $\beta$, as usual.

We want a small refinement of this bound. Suppose that we know in addition that each $Y_m$ is non-zero with probability at most $p\in(0,1)$. Then, conditioning upon the events that some $k$ of $Y_m$ have any chance to be non-zero, we get the bound $$ P(|Y|\ge t)\le C\sum_{k=1}^n e^{-c\frac{t^2}{ks^2}}{n\choose k}p^k(1-p)^{n-k}\,. $$ Using the inequality $\frac 1k\ge 2\beta-\beta^2 k$, we can estimate the RHS by $$ Ce^{-2c\frac{t^2}{s^2}\beta}\sum_{k=1}^n {n\choose k}e^{c\beta^2\frac{t^2}{s^2}k}p^k(1-p)^{n-k} =Ce^{-2c\frac{t^2}{s^2}\beta}\left[1+p(e^{c\beta^2\frac{t^2}{s^2}}-1)\right]^n\,. $$ for any $\beta>0$ we want.

Now let us look at the distribution of our random variable $X$ assuming that $E|X|^q=1$. For every $p$, we can take the set $F$ of probability $p$ on which it attains the largest values and split $X$ as $X'+X''$ where $X'=X$ outside $F$ and $X'=\frac 1p E(X\chi_F)$ on $F$. If $EX=0$, then $EX'=EX''=0$, $E|X'|^q\le E|X|^q$, $E|X''|^q\le CE|X|^q$ but $X''$ is not zero only with probability $\le p$ and $|X'|\le s$ where $s^qp=1$ (by Jensen). We can apply this trick successively with $p=2^{-r}, \log_2 n\ge r\ge 0$ and get the decomposition of $X$ into the sum of mean $0$ random variables $Z+\sum_{r=0}^{\log_2n}{X_r}$ where $Z$ is different from $0$ with probability about $\frac 1n$ and $E|Z|^q\le C$, while for each $r$, we have $|X_r|\le s_r=2^{r/q}$ and $X_r$ is not zero only with probability $p_r=2^{-r+1}$.

Now it will be enough to treat each $X_r$ and $Y$ separately. The exact nature of the discrete Fourier transform does not matter. All we need to know is that we are interested in the maximum of $n$ linear forms of $n$ iid copies of our random variables with coefficients not exceeding $\frac 1{\sqrt n}$.

Let's start with $n$ iid copies of $Z$. Let $N$ be the number of non-zero values among $Z_1, \dots, Z_n$. Notice that it is the sum of $n$ iid Bernoulli random variables each of which is $1$ with probability $\frac 1n$, so $Ee^N=(1+\frac en)^n\le e^e$. In particular, any fixed moment of $N$ is bounded by some constant.

We have for $q>2$, $$ \frac 1n \left(\sum_m|Z_m|\right)^2\le \frac 1n N^{2-\frac 2q}\left[\sum_m |Z_m|^q\right]^{2/q} $$ so, using this crude bound and the Holder inequality, we get $$ E(\max(Z-\text{forms})^2)\le\frac 1n (EN^{\text{something}})^{1-\frac 2q}\left[E\sum_m |Z_m| ^q\right]^{2/q}\le Cn^{\frac 2q-1}\to 0 $$ as $n\to\infty$. Thus this part is negligible for large $n$.

Now let us fix $r$ and consider $X_r$. Notice that we can find $\delta>0$ such that $\frac 1q +\delta<\frac 12$. Then if we change the notation $X_r$ to $2^{-\delta r}X$, $p_r=2^{-r+1}$ to $p$, and $s_r=2^{r/q}$ to $2^{-\delta r}s$, we shall still have $s\le n^{\frac 12- \varepsilon}$ with some $\varepsilon>0$ and $p\le s^{-2}$ for all $r\le\log_2n$. The extra exponential factor $2^{-\delta r}$ we introduced is strong enough to enable us to consider each such $X$ separately and just to get a uniform bound of order $\log n$ for the expectations of the squared maximum of $n$ linear $X$-forms $L_j$.

Now comes the trick: in order to show that $E(\max_{1\le j\le n}|L_j|)^2\le C^2\log n+O(1)$, it suffices to show that for each individual $j$, we have $$ E(|L_j|^2-C^2\log n)_+=\int_{C\sqrt{\log n}}^\infty 2tP(|L_j|>t)\,dt\le \frac Cn\,. $$
However, we have the refined bound for the probability in question and, taking into account the $\sqrt n$ in the denominator, changing $\beta$ to $\beta/n$, and using the bound $p\le s^ {-2}$, we can rewrite it as $$ P(|L_j|>t)\le Ce^{-2c\frac{t^2}{s^2}\beta}\left[1+s^{-2}(e^{c\beta^2\frac{t^2}{ns^2}}- 1)\right]^n $$

We have to choose the optimal $\beta=\beta(t)$. There are two cases to consider:

Case 1: $t^2s^2\le n$.

In this case we can take $\beta$ a small multiple of $s^2$ (the difference of the exponent and $1$ can be treated like a linear function in this range) and get the estimate $e^{-c't^2}$, which is as good as if we were dealing with (sub)Gaussian variables.

Case 2: $t^2s^2>n$. In this case we still want to stay in the linear range for the exponent in the parentheses, so we are forced to take $\beta$ a small multiple of $\frac{s\sqrt n}t$. Fortunately, it still gives the bound $e^{-c'\frac{\sqrt n}{s}t}$ and even if we integrate it against $t$ from $1$, we still obtain something like $e^{-c'\frac{\sqrt n}s}\le e^{-c'n^\varepsilon}$, which is much smaller than what we need for large $n$.

$\endgroup$
8
  • $\begingroup$ I'd like to ask for a clarification about the decomposition of $X$. Define $F_0=\{|X|^q\le 1\}$, $F_r=\{2^{r-1}<|X|^q\le 2^r\}$ for $r\ge1$, $p_r=P(F_r)$ and $G_r=\{|X|^q>2^r\}$ for $r\ge0$. Denote $$ X_r =(X-p_r^{-1}E[X\chi_{F_r}])\chi_{F_r}, \quad Z =X\chi_{G_{\log_2n}}+\sum_{r=0}^{\log_2n}p_r^{-1}E[X\chi_{F_r}]\chi_{F_r} $$ so that $X=Z+\sum_{r=0}^{\log_2n}X_r$. If I use such a decomposition, $Z$ is never equal to $0$. I can modify this decomposition so that $P(Z\ne0)\le n^{-1}$, but then $X_r$'s are not equal to $0$. Could you please give a hint? $\endgroup$
    – Cm7F7Bb
    Nov 21, 2018 at 15:49
  • 1
    $\begingroup$ @Cm7F7Bb What I'm doing is slightly different. I take the set of largest values of $|X|$ of fixed probability and replace $X$ on it by its average to get $X'$. I also start with the largest level, so the first thing I get is $Z$ (so I write $X=Z+X'$). Then I apply this trick to $X'$ to chop off $X_{\log_2 n}$ and so on. $\endgroup$
    – fedja
    Nov 21, 2018 at 16:01
  • $\begingroup$ I’m struggling to understand why it is enough to treat each $X_r$ separately. We need to show that $$ E\max_{1\le j\le n}\Bigl|\sum_{t=1}^na_{jnt}\sum_{r=0}^{\log_2n}X_{rt}\Bigr|^2=O(\log n), $$ where $|a_{jnt}|\le n^{-1/2}$. I find it a little bit confusing when the notation is changed from $X_r$ to $2^{-\delta r}X$ since $X$ is a random variable that we want to decompose. Could you please explain in more detail how the extra exponential factor $2^{-\delta r}$ works and why it is enough to consider each $X_r$ separately? I’d greatly appreciate that. $\endgroup$
    – Cm7F7Bb
    Nov 23, 2018 at 10:49
  • $\begingroup$ @Cm7F7Bb You just show that the expectation of the maximum for $X_r$ is at most $C2^{-2\delta r}\log n$ and then use the triangle inequality. $\endgroup$
    – fedja
    Nov 23, 2018 at 11:37
  • $\begingroup$ Could you please explain in more detail how the set $F$ is chosen? We have that $P(F)=p$ and $F=\{|X|>s\}$ since $|X'|\le s$. What is the relation between $s$ and $p$? I can only obtain that $s^qp\le 1$ using Markov's inequality and the assumption that $E|X|^q=1$. However, it is stated in the proof that $s^qp=1$ which is obtained using Jensen's inequality. Could you explain in more detail? I would greatly appreciate that. $\endgroup$
    – Cm7F7Bb
    Nov 28, 2018 at 10:24

Your Answer

By clicking “Post Your Answer”, you agree to our terms of service and acknowledge you have read our privacy policy.

Not the answer you're looking for? Browse other questions tagged or ask your own question.